Determine Series' Convergence or Divergence

In summary, the conversation discusses the convergence of the series ##\sum _{n=1}ne^{-n}## using the ratio and integral tests. The ratio test determines that the series converges absolutely, but the integral test is inconclusive. The conversation then discusses using l'Hospital's rule to find the limit of ##\frac{n}{e^n}##, which is necessary for the integral test. After some confusion, it is determined that the limit is ##\frac{2}{e}##, confirming that the series converges.
  • #1
knowLittle
312
3

Homework Statement


##\sum _{n=1}ne^{-n}##


Homework Equations


Ratio Test
Integral Test


The Attempt at a Solution


I know that by the ratio test, it converges absolutely. But, I am unable to determine its convergence through the integral test . Could someone help? I thought that the integral test would be best, since e is used.

The ratio test gives me 1/e approx 0.367879 <1; therefore, it converges absolutely.

Now, the integral test is my concern. I believe I have done all correct steps, but the result shows that it diverges.
##\lim _{b\rightarrow \infty }\int _{1}^{b}\dfrac {n} {e^{n}}dn##

Integrating by parts with: u=n; du=dn; dv=(e^-n) dn; v=-e^-n
## \left[ -e^{-n}\left( n\right) +-e^{-n}\right] _{1}^{\infty }##
The final result is:
##-\dfrac {\infty } {e^{\infty} }-\dfrac {1} {e^{\infty }}+\dfrac {2} {e} ## and it diverges. What am I doing wrong?
Thank you.
 
Physics news on Phys.org
  • #2
knowLittle said:

Homework Statement


##\sum _{n=1}ne^{-n}##


Homework Equations


Ratio Test
Integral Test


The Attempt at a Solution


I know that by the ratio test, it converges absolutely. But, I am unable to determine its convergence through the integral test . Could someone help? I thought that the integral test would be best, since e is used.

The ratio test gives me 1/e approx 0.367879 <1; therefore, it converges absolutely.

Now, the integral test is my concern. I believe I have done all correct steps, but the result shows that it diverges.
##\lim _{b\rightarrow \infty }\int _{1}^{b}\dfrac {n} {e^{n}}dn##

Integrating by parts with: u=n; du=dn; dv=(e^-n) dn; v=-e^-n
## \left[ -e^{-n}\left( n\right) +-e^{-n}\right] _{1}^{\infty }##
The final result is:
##-\dfrac {\infty } {e^{\infty} }-\dfrac {1} {e^{\infty }}+\dfrac {2} {e} ## and it diverges. What am I doing wrong?
Thank you.

What is ##\displaystyle \lim_{n \rightarrow \infty} \frac{n}{e^n}##? You will need to use l'Hospital's rule!
 
  • #3
Wow. Thank you.
 
  • #4
My integral after L'Hospital doesn't agree with answers online. Could you tell me what's wrong?
##\lim _{b\rightarrow \infty }\int _{1}^{b}\dfrac {n} {e^{n}}dn##

##\int _{1}^{b}\dfrac {1} {en}dn=\left[ -e^{-n}\right] _{1}^{b}=-\dfrac {1} {e^{\infty }}+\dfrac {1} {e}##
, the first term to the left is e^n
 
  • #5
You need to find the limit of [itex]\frac{1}{e^n}[/itex] and not its integral.
 
  • #6
The limit of (1/ e^n) as n approaches infinity is 0. The test for divergence is inconclusive.

I'm trying to find the determination of convergence or divergence through the integral test.
 
Last edited:
  • #7
OK, I've worked it out, and the limit should be: [itex]\frac{2}{e}[/itex]
 
Last edited:
  • #8
Yeah, that's what it's shown online.
I get 1/e. My procedure is above. Could you help?
 
  • #9
This step is correct:
[tex]\left[ -e^{-n}\left( n\right) -e^{-n}\right] _{1}^{\infty }[/tex]
Let b=∞, then,
[tex]\left[ -e^{-n}\left( n\right) -e^{-n}\right] _{1}^{b}[/tex]
This gives:
[tex]\left[ \left( \frac{-b}{e^b} -e^{-b}\right) - \left( \frac{-1}{e} -e^{-1}\right) \right][/tex]
Now, what you have to find is the limit as b→∞
[tex]\lim_{b→∞} \left[ \left( \frac{-b}{e^b} -e^{-b}\right) - \left( \frac{-1}{e} -e^{-1}\right) \right][/tex]
But the limit of the following requires using L'Hopital's rule,
[tex]\lim_{b→∞} \frac{-b}{e^b} = \lim_{b→∞} \frac{-1}{e^b}[/tex]
You can then work out the rest:
[tex]\lim_{b→∞} \left[ \left( \frac{-1}{e^b} - \frac{1}{e^b}\right) - \left( \frac{-1}{e} - \frac{1}{e}\right) \right][/tex]
 
  • #10
Thanks, it's what I suspected. This L'Hospital rule keeps coming and coming.
 
  • #11
You had the answer in your first post! The first term goes to zero by l'Hospital's, the second goes to zero, and the final term is just a constant which was ##\frac{2}{e}##.
 
  • #12
I know, I didn't know I could use L'Hospital in the middle of an integration by parts.
Thanks.
 

What is the definition of "convergence" in a series?

Convergence in a series refers to the behavior of the terms in a sequence as the number of terms increases. If the terms approach a finite number as the number of terms increases, the series is said to converge.

How can you determine if a series is convergent or divergent?

One way to determine the convergence or divergence of a series is by using the limit comparison test, which compares the given series to a known series with known convergence or divergence. Another method is the ratio test, which compares the ratio of consecutive terms in the series to determine convergence or divergence.

What is the difference between absolute and conditional convergence?

Absolute convergence refers to a series in which the sum of the absolute values of the terms converges to a finite number. Conditional convergence refers to a series in which the sum of the terms converges, but the sum of the absolute values of the terms does not.

Can a series have both absolute and conditional convergence?

Yes, a series can have both absolute and conditional convergence. This occurs when the series is absolutely convergent, meaning the sum of the absolute values of the terms converges, but not unconditionally convergent, meaning the sum of the terms themselves also converges.

What is the significance of a series' convergence or divergence?

The convergence or divergence of a series is important because it determines whether the series has a finite sum or not. Convergent series have a finite sum, while divergent series do not. This can be useful in real-life applications such as calculating the total cost of a project or determining the value of an investment.

Similar threads

  • Calculus and Beyond Homework Help
Replies
2
Views
189
  • Calculus and Beyond Homework Help
Replies
2
Views
711
  • Calculus and Beyond Homework Help
Replies
1
Views
259
  • Calculus and Beyond Homework Help
Replies
34
Views
2K
  • Calculus and Beyond Homework Help
Replies
4
Views
311
  • Calculus and Beyond Homework Help
Replies
3
Views
418
  • Calculus and Beyond Homework Help
Replies
7
Views
2K
  • Calculus and Beyond Homework Help
Replies
1
Views
655
  • Calculus and Beyond Homework Help
Replies
6
Views
1K
  • Calculus and Beyond Homework Help
Replies
5
Views
489
Back
Top